Jump to content

Hijack from intuitive model of a particle wave


Butch

Recommended Posts

Perhaps if the y or z axis was shifted by 90°? Then your photon becomes somewhat of a screw.

 

I am not sure what you mean by rotating an axis through 90o.

 

If the electric and magnetic fields are 180o out of phase (in antiphase) then the wave is travelling in the opposite direction (from right to left).

 

The E vector certainly does take on a helical path if the wave is circularly polarised.

 

However that would be off topic since the OP defines a plane wave.

 

The energy or intensity is proportional to the square of E.

 

This theory also has nothing to do with photons.

 

 

Edguy99 post#1

A photon is a plane wave travelling through space at the speed of light.

 

Can't see this as a valid interpretation of a plane wave.

Edited by studiot
Link to comment
Share on other sites

 

I am not sure what you mean by rotating an axis through 90o.

 

If the electric and magnetic fields are 180o out of phase (in antiphase) then the wave is travelling in the opposite direction (from right to left).

 

The E vector certainly does take on a helical path if the wave is circularly polarised.

 

However that would be off topic since the OP defines a plane wave.

 

The energy or intensity is proportional to the square of E.

 

This theory also has nothing to do with photons.

 

 

Can't see this as a valid interpretation of a plane wave.

If one axis creating the plane is shifted by 90° the plane becomes a helix, its energy remains constant as does frequency. Just a thought...
Link to comment
Share on other sites

If one axis creating the plane is shifted by 90° the plane becomes a helix, its energy remains constant as does frequency. Just a thought...

 

 

studiot post#13

The E vector certainly does take on a helical path if the wave is circularly polarised.

 

Did you miss this?

 

I emphasized the 'if' to remind you that the OP specified a plane wave, not a circularly polarised one.

 

It's there, writ large in post#1.

 

Remember also that if you rotate one vector (you rotate the E and B vectors not the axes themselves) you must rotate the other one to maintain a proper relationship.

 

But all this is off topic.

 

In a plane wave, the E vector and the B vector are separately plane waves at right angles.

The phase angle between the E wave and the B wave is either zero or pi depending upon the direction of travel of the wave.

(This is why some authors show the E wave in phase with the B wave and others show it in antiphase)

In all cases the direction of travel corresponds to the cross product of the E and B vectors.

 

Further in a plane wave the E vector and the B vector are identical at every point in the plane, at right angles to the direction of travel.

Link to comment
Share on other sites

 

 

Did you miss this?

 

I emphasized the 'if' to remind you that the OP specified a plane wave, not a circularly polarised one.

 

It's there, writ large in post#1.

 

Remember also that if you rotate one vector (you rotate the E and B vectors not the axes themselves) you must rotate the other one to maintain a proper relationship.

 

But all this is off topic.

 

In a plane wave, the E vector and the B vector are separately plane waves at right angles.

The phase angle between the E wave and the B wave is either zero or pi depending upon the direction of travel of the wave.

(This is why some authors show the E wave in phase with the B wave and others show it in antiphase)

In all cases the direction of travel corresponds to the cross product of the E and B vectors.

 

Further in a plane wave the E vector and the B vector are identical at every point in the plane, at right angles to the direction of travel.

I think you misunderstood, shift the waveform on one axis so that they are 90° out of phase rather than 180° or 0°.
Link to comment
Share on other sites

I think you misunderstood, shift the waveform on one axis so that they are 90° out of phase rather than 180° or 0°.

 

Can you demonstrate how that might be possible?

 

I note you are now saying shift not rotate.

 

The maxima of the electric and magnetic vectors are interdependent so must coincide along the axis of shift.

Link to comment
Share on other sites

 

Can you demonstrate how that might be possible?

 

I note you are now saying shift not rotate.

 

The maxima of the electric and magnetic vectors are interdependent so must coincide along the axis of shift.

The electric creates the magnetic which lags by 90 degrees which is a property of induction. A photon is after all an electromagnetic system. This would not produce a helical structure, but rather a helical path.

A thought just occurred to me, would such a path create inertia?

Edited by Butch
Link to comment
Share on other sites

The electric creates the magnetic which lags by 90 degrees which is a property of induction. A photon is after all an electromagnetic system. This would not produce a helical structure, but rather a helical path.

A thought just occurred to me, would such a path create inertia?

 

No

 

We can derive this directly from Faraday's Law of induction which states:

 

The induced voltage is directly proportional to the time rate of decrease of the magnetic field.

 

[math]\frac{{\partial E}}{{\partial x}} \propto - \frac{{\partial B}}{{\partial t}}[/math]
Now the OP shows a sinusoidal E wave and B wave and using the same axes we work as follows
Let t be the time,
[math]\omega [/math] is the angular frequency of the wave
k a constant
[math]\varphi [/math] is the yet to be determined phase difference between the E wave and the B wave.
The E wave may be represented as
[math]{E_{\left( {x,t} \right)}} = {E_0}\cos \left( {\omega t - kx} \right)[/math]
and the B wave as
[math]{B_{\left( {x,t} \right)}} = {B_0}\cos \left( {\omega t - kx + \varphi } \right)[/math]
Differentiate the E wave with respect to distance, x
[math]\frac{{\partial E}}{{\partial x}} = k{E_0}\sin \left( {\omega t - kx} \right)[/math]
and the B wave with respect to time, t and taking the engative
[math] - \frac{{\partial B}}{{\partial t}} = \omega {B_0}\sin \left( {\omega t - kx + \varphi } \right)[/math]
Faraday's Law tells us these must be equal everywhere so
If k is positive, ie the wave is proceeding in the positive x direction, then
[math]\varphi = 0[/math]
If k is negative, ie the wave is proceeding in the negative x direction, then
[math]\varphi = \pi [/math]

These are the only allowable values that fit the equations.

Link to comment
Share on other sites

 

No

 

We can derive this directly from Faraday's Law of induction which states:

 

The induced voltage is directly proportional to the time rate of decrease of the magnetic field.

 

[math]\frac{{\partial E}}{{\partial x}} \propto - \frac{{\partial B}}{{\partial t}}[/math]

 

Now the OP shows a sinusoidal E wave and B wave and using the same axes we work as follows

 

Let t be the time,

 

[math]\omega [/math] is the angular frequency of the wave

k a constant

[math]\varphi [/math] is the yet to be determined phase difference between the E wave and the B wave.

 

The E wave may be represented as

 

 

[math]{E_{\left( {x,t} \right)}} = {E_0}\cos \left( {\omega t - kx} \right)[/math]

 

and the B wave as

 

 

[math]{B_{\left( {x,t} \right)}} = {B_0}\cos \left( {\omega t - kx + \varphi } \right)[/math]

 

Differentiate the E wave with respect to distance, x

 

[math]\frac{{\partial E}}{{\partial x}} = k{E_0}\sin \left( {\omega t - kx} \right)[/math]

 

and the B wave with respect to time, t and taking the engative

 

 

 

[math] - \frac{{\partial B}}{{\partial t}} = \omega {B_0}\sin \left( {\omega t - kx + \varphi } \right)[/math]

 

Faraday's Law tells us these must be equal everywhere so

 

If k is positive, ie the wave is proceeding in the positive x direction, then

[math]\varphi = 0[/math]

 

If k is negative, ie the wave is proceeding in the negative x direction, then

 

[math]\varphi = \pi [/math]

 

These are the only allowable values that fit the equations.

This shows a phase relationship of 90° between voltage and current and 0° between current and field. Edited by Butch
Link to comment
Share on other sites

This shows a phase relationship of 90° between voltage and current and 0° between current and field.

 

Perhaps you would like to show mathematically how this claim follows from the mathematics?

 

While you are at it please explain your use of the word 'current' in this context?

Edited by studiot
Link to comment
Share on other sites

 

Perhaps you would like to show mathematically how this claim follows from the mathematics?

 

While you are at it please explain your use of the word 'current' in this context?

Ohms law states E=IR since we have no current and no resistance we are left with the field and the force producing it, Faraday says induction is a relationship between the the electromagnetic field and the electromotive force it produces. Lenzs law gives us hysteresis, which in a sinusoidal wave produces a current lag of 90° since field strength is proportional to current, it would seem to follow that field would lag force. This is all conjecture on my part, just thought it might help.
Link to comment
Share on other sites

Ohms law states E=IR since we have no current and no resistance we are left with the field and the force producing it, Faraday says induction is a relationship between the the electromagnetic field and the electromotive force it produces. Lenzs law gives us hysteresis, which in a sinusoidal wave produces a current lag of 90° since field strength is proportional to current, it would seem to follow that field would lag force. This is all conjecture on my part, just thought it might help.

 

 

So no mathematics then? Just guesswork.

Link to comment
Share on other sites

Guess work, conjecture... Pretty much the same.

I am not promoting it as fact, just a thought.

 

!

Moderator Note

Two problems here. One is that you are "guessing" in someone else's thread, which is hijacking. The other is that without the math, it's not enough. Speculations requires more than guesswork

Link to comment
Share on other sites

I appologize, I'm afraid I wasn't very clear about the final model that I was trying to propose.

 

Lets take another look at the origional particle wave model

post-115209-0-02422200-1483881332_thumb.png

 

 

and the helix model:

post-115209-0-37573200-1483881365_thumb.gif

Click on the image to show the animation.

 

Differences you should ignore:

We need to ignore the axis and color orientations. Also the arrows showing the directions the waves are pointing in the first model are missing in the second model. I appologize, I had to make do with wharever gifs that I could find in the Wikipedia library that best fit the models since I dont have a PC to create the gifs.

 

Differences that are intented:

A key difference is that the particle is always there located at the tip of the arrow. For a light wave the particle is a photon, bit it can represent any particle in free space.

 

We can see both the electric wave and perpendicular magnetic wave as orthoganal projections of the helix model. Therefore nothing much has been changed about what we know about the old model, However, an invisible half was added to the new model that makes a little more sense about the particle/wave itself but explains much more about duality and QM

 

More will be clarified in part 2 the intuitive model for QM

 

Also notice that instead of being 180⁰ phase shifted, they are 90⁰ phase shifted as they used to be defined which I am certain back around the late 1990s. I actually recall some of the pictures when they started changing, which I presume was to make it appear more like they were exchanging energy between them.

 

post-115209-0-05957400-1483881387_thumb.gif

 

Whats missing that we need to imagine is there:

Imagine a second helix rotating in the same ccw direction but with the opposite phase to the first helix so that together you get a Dual Helix.

 

Instead of a single particle, there would be a particle and an anti-particle locked in a dual orbit with the following properties: equal mass, opposite charge, in 180⁰ phase shift, and Hidden. Notice while the anti-particle is hidden due to being 180⁰ phase shifted, the EM fields would still interact since fields can cross parallel dimensions, but particles cannot.

 

If we took the orthoganol projections of the Dual Helix (imagine both helixes are visible) then the electric wave would be a differential wave pair and perpendicular to the magnetic wave thar would also be a differential wave pair.

 

post-115209-0-02163000-1483881440_thumb.png

(again ignore the labels and colors)

 

Differential pairs have a zero net energy and are self referencing in the y axis (no gnd needed) and self referencing in the x axis (no clock needed)

 

All four waves in the two pairs would be moving in the same direction.

 

 

The HSDD Analog:

post-115209-0-23747000-1483881456_thumb.png

Above represents a pair of differential copper traces over a return path plane with fiberglass material between the bottom of the traces and the top surface of the ground plane in a microsrtip configuration.

 

Below is a model for showing the top view looking down at the trace pairs. The copper traces are grey, the balck dots are electrons and the arrows are the EM field.

post-115209-0-59214300-1483881479.gif

 

Below Imagine the arrows are oscilating between up and down, then outline of the arrows represents the orthogonal projection of the differential wave pair in 2D

post-115209-0-76051900-1483881497.gif

 

Now imagine the arrows are rotating perpendicular to the screen. The outline of the arrows now represents a dual helix in 3D

post-115209-0-63318900-1483881514.gif

 

Another duality issue I forgot to explain last time:

particle passing both slits at once:

The wave behavior in the dual slit experiment had implied that a single photon had to be going through both slits at once in order to know when waves would cancel each other out.

 

How dual helix model can explain it:

If you look up the silicon droplet experiment they found that pairs of drops in a dual orbit passing through a slit would change their angle such that they created an inteferance pattern without needing to interfere with another particle.

 

The same thing happens with the particle/antiparticle pairs that are in a dual orbit

Edited by TakenItSeriously
Link to comment
Share on other sites

Instead of a single particle, there would be a particle and an anti-particle locked in a dual orbit with the following properties: equal mass, opposite charge, in 180⁰ phase shift, and Hidden. Notice while the anti-particle is hidden due to being 180⁰ phase shifted, the EM fields would still interact since fields can cross parallel dimensions, but particles cannot.

 

 

Be sure to include some evidence for this, and/or ways it can be tested.

Link to comment
Share on other sites

Instead of a single particle, there would be a particle and an anti-particle locked in a dual orbit with the following properties: equal mass, opposite charge, in 180⁰ phase shift, and Hidden. Notice while the anti-particle is hidden due to being 180⁰ phase shifted, the EM fields would still interact since fields can cross parallel dimensions, but particles cannot.

 

Be sure to include some evidence for this, and/or ways it can be tested.

I guess I was thinking that all fields were considered to be able to exist between dimensions but perhaps thats only gravity fields?

 

how about:

As evidence for this the particle/antiparticle pair creates a dipole moment which could explain why charged particles have intrinsic spin magnetic moments:

 

μB = eħ/(2me·c)

where

e is the elementary charge,

ħ is the reduced Planck constant,

me is the electron rest mass and

c is the speed of light.

 

 

 

 

 

​​

Link to comment
Share on other sites

I guess I was thinking that all fields were considered to be able to exist between dimensions but perhaps thats only gravity fields?

how about:

As evidence for this the particle/antiparticle pair creates a dipole moment which could explain why charged particles have intrinsic spin magnetic moments:

μB = eħ/(2me·c)

where

e is the elementary charge,

ħ is the reduced Planck constant,

me is the electron rest mass and

c is the speed of light.

​​

Is there evidence of parallel dimensions?

Link to comment
Share on other sites

Is there evidence of parallel dimensions?

Sorry, I'm still not sure what it is your asking for.

 

This source explains how according to the Big Bang Theory, every particle in the universe was created as a matter/antimatter entangled pair, another words a universe of matter and a mirrored universe of antimatter.

http://www.physicsoftheuniverse.com/topics_bigbang_antimatter.html

 

I've alredy given an abundance of evidence in the origional thread here:

http://www.scienceforums.net/topic/101660-an-intuitive-particle-wave-model/

 

Following is a Summary of the Evidence:

  • A hidden antimatter universe explains the missing antimatter created by the big bang.
  • A matter/antimatter entangled universe solves entanglements apparent spooky information at a distance, problem. Alice/anti-Alice and Bob/anti-Bob observers observer up and down spin states.
  • Based on the silicon drop experiment, particles in a dual orbit bassing through the slits, create the band like dispersal without needing a single particle to pass through both slits.
  • Particle/Wave duality is due to entangled pairs causing the wave effect and broken entanglement causing the particle effect.
  • Observers causing the wave to particle transition is caused by observers breaking entanglement.
  • Heisenberg Uncertainty is caused by the position/momentum information split between the two universes.
  • Entangled +/- particles in a dual orbit creates a dipole moment that can explain intrinsic magnetic moments of charged particles.
  • Dual helix model creates differential electric waves perpendicular to differential magnetic waves which results in a 0 net energy state, or waves that may self propagate through a vacuum indefinitely.
  • Differential waves negate any need for a ground reference or a clock reference.
Edited by TakenItSeriously
Link to comment
Share on other sites

A matter/antimatter entangled universe solves entanglements apparent spooky information at a distance, problem. Alice/anti-Alice and Bob/anti-Bob observers observer up and down spin states.

 

In typical entanglement experiment, there is no antimatter created, just photons...

Link to comment
Share on other sites

[*]A matter/antimatter entangled universe solves entanglements apparent spooky information at a distance, problem. Alice/anti-Alice and Bob/anti-Bob observers observer up and down spin states.

In typical entanglement experiment, there is no antimatter created, just photons...

The matter/antimatter entanglement I am referring to is the matter/antimatter created by the BB which I havent explained fully yet as that is in Part 2 An Intuitive Model for Quantum Mechanics.

 

I will probably merge part 1 & part 2 when I post part 2.

 

Unfortunately this confusion is a consequence of the fact that this is a converging model for a TOE as I posted at the top of the origional thread.

 

Therefore, I need to provide rather long replies to short questions that needs to cover more and more of the unposted sections. It's difficult to explain, and even more difficult to write in the first place.

 

Hopefully, once I get it all posted, with links it wont be so confusing.

Link to comment
Share on other sites

<p>

Sorry, I'm still not sure what it is your asking for.

 

This source explains how according to the Big Bang Theory, every particle in the universe was created as a matter/antimatter entangled pair, another words a universe of matter and a mirrored universe of antimatter.

http://www.physicsoftheuniverse.com/topics_bigbang_antimatter.html

 

I've alredy given an abundance of evidence in the origional thread here:

http://www.scienceforums.net/topic/101660-an-intuitive-particle-wave-model/

 

Following is a Summary of the Evidence:]

None of this counts as evidence. Not here or in the original threads. These are all assertions. In point of detail your post above reflects connecting dots based on lack of understanding of entanglement, Cosmology and the nature of antimatter and matter.

 

I have yet to see any evidence that supports your claims.

 

For starters particle/wave duality has nothing to do with entanglement nor the Heisenburg uncertainty principle. These are inherent in every particle. Matter or antimatter.

 

The main difference between the two is simply opposite charge. Nothing you have posted accounts for leptogenesis nor baryogenesis. That requires detailed mathematics. Which you have yet to provide correctly. Modelling electromagnetic fields is insufficient for this. Its a completely different dynamic. The only similarity is helicity.

 

As Swansont mentioned you haven't shown any evidence of an antimatter dimension that we cannot detect. All particles contribute to temperature. We can see antimatter being created and annihilating from stars and we know antimatter is being created by our sun.

 

Our planet is continously bombarded by antimatter emitted by our sun. So how can it exist in a seperate universe overlapping our own let alone being spatially seperated.?

 

What evidence do you have of a hidden antimatter universe? Quite frankly you have a claim. Not evidence.

 

Do you even understand the nature of a particle? All particles are field excitations. Particle/wave duality are simply different aspects of these excitations. So is all quantum numbers including charge and spin. Matter/antimatter included. The Dirac equations mentioned in the physicsoftheuniverse link you posted predicted the existence of antimatter via these field relations. Perhaps you should study them instead of making baseless assertions.

 

The biggest problem is however misinterpreting the Feynman-Stuckelberg interpretation. This is specifically the origin of the backward in time treatment for antiparticles. However pop media definetely doesn't portray this correctly lmao. Correctly it is particle motion in coordinate time. Not proper time.

 

There is no causality violation involved as it is a mathematical treatment of time reversal symmetry describing the probability density distribution.

 

A more accurate interpretion is the emission absorption of a negative energy particle with momentum p as the absorption emission of a positive energy antiparticle with momentum -p.

 

In other words a vector momentum operator under a reversal of sign. Either positive or negative direction. Which can equivalently be modelled as time reversal to remove negative mass.

 

This is what is shown under CPT symmetry. Not that antiparticles are truly travelling backward in proper time. That would be a causality violation.

 

In essence the time reversal symmetry is identical to a vector direction. Travelling away from or towards a coordinate. We have the same time reversal symmetry in the Lorentz group. This does not mean the spaceship travels backward in time under time reversal symmetry in relativity. Neither does a particle travel backward in time under the Feynman-Stuckelberg interpretation. Its vector direction is simply modelled as time reversal in coordinate time not proper time.

Edited by Mordred
Link to comment
Share on other sites

Create an account or sign in to comment

You need to be a member in order to leave a comment

Create an account

Sign up for a new account in our community. It's easy!

Register a new account

Sign in

Already have an account? Sign in here.

Sign In Now
×
×
  • Create New...

Important Information

We have placed cookies on your device to help make this website better. You can adjust your cookie settings, otherwise we'll assume you're okay to continue.